Difference between revisions of "2021 AMC 10A Problems/Problem 2"

(Blanked the page)
(Tag: Blanking)
Line 1: Line 1:
 +
==Problem 2==
 +
Portia's high school has <math>3</math> times as many students as Lara's high school. The two high schools have a total of <math>2600</math> students. How many students does Portia's high school have?
  
 +
<math>\textbf{(A)} ~600 \qquad\textbf{(B)} ~650 \qquad\textbf{(C)} ~1950 \qquad\textbf{(D)} ~2000\qquad\textbf{(E)} ~2050</math>
 +
 +
==Solution==
 +
The following system of equations can be formed with <math>p</math> representing the number of students in Portia's high school and <math>l</math> representing the number of students in Lara's high school.
 +
<cmath>p=3q</cmath>
 +
<cmath>p+q=2600</cmath>
 +
Substituting <math>p</math> with <math>3q</math> we get <math>4q=2600</math>. Solving for <math>q</math>, we get <math>q=650</math>. Since we need to find <math>p</math> we multiply <math>650</math> by 3 to get <math>p=1950</math>, which is $\boxed{\text{C}}

Revision as of 14:39, 11 February 2021

Problem 2

Portia's high school has $3$ times as many students as Lara's high school. The two high schools have a total of $2600$ students. How many students does Portia's high school have?

$\textbf{(A)} ~600 \qquad\textbf{(B)} ~650 \qquad\textbf{(C)} ~1950 \qquad\textbf{(D)} ~2000\qquad\textbf{(E)} ~2050$

Solution

The following system of equations can be formed with $p$ representing the number of students in Portia's high school and $l$ representing the number of students in Lara's high school. \[p=3q\] \[p+q=2600\] Substituting $p$ with $3q$ we get $4q=2600$. Solving for $q$, we get $q=650$. Since we need to find $p$ we multiply $650$ by 3 to get $p=1950$, which is $\boxed{\text{C}}